Calcolo di un integrale

Messaggioda Isaac888 » 14/11/2022, 14:57

Salve, devo integrare questa bestiolina:
$$I:=\frac{1}{2\pi}\int_0^\pi{\frac{1-r^2}{1+r^2-2rcos(\theta-\phi)}}d\phi$$
dove $0<r<1$ è costante, $\theta\in[0,2\pi[$ costante e la variabile di integrazione $\phi\in[0,2\pi[$.
Avevo pensato di vederla così: se $a:=\frac{2r}{1+r^2}$ (dove $0<a<1$, per $r\ne 1$) allora
$$I=\frac{1-r^2}{2\pi(1+r^2)}\int_0^{\pi}{\frac{1}{1-a\cdot cos(\theta-\phi)}}d\phi$$
Chiamando $$J:=\int_0^{\pi}{\frac{1}{1-a\cdot cos(\theta-\phi)}}d\phi \implies I=\frac{1-r^2}{2\pi(1+r^2)}J$$
così mi risolvo $J$.
Ora, ci sono almeno due strade per risolvere questo integrale. Una è quella di calcolare la primitiva e valutarla negli estremi di integrazione, l'altra (che è quella che voglio usare) è usare il teorema dei residui. Con questa ultima via non ne sto venendo a capo però. Qualcuno mi può aiutare per favore? Non capisco dove sto sbagliando. Non riesco ad ottenere una dipendenza da $\theta$.

I miei calcoli:
Testo nascosto, fai click qui per vederlo
Faccio una sostituzione per portare gli estemi in $[0,2\pi]$: se $t=2\phi \implies d\phi=\frac{dt}{2}$. Così il mio integrale diventa:
$$\frac{1}{2}\int_0^{2\pi}{\frac{dt}{1-a\cdot cos(\theta-\frac{t}{2})}}$$
A questo punto se chiamo $z:=e^{i(\theta-\frac{t}{2})$ allora $cos(\theta - \frac{t}{2})=\frac{z+\bar{z}}{2}$, mentre $dt=-\frac{2}{i}\frac{dz}{z}$. Così sostituendo tutto ottengo:
$$\frac{4}{ia}\oint_{\gamma}\frac{dz}{z^2-\frac{2}{a}z+1}=\frac{4}{ia}\oint_{\gamma}f(z)dz$$
dove $f(z):=\frac{1}{z^2-\frac{2}{a}z+1}$, che è una funzione razionale senza poli sulla circonferenza unitaria parametrizzata dalla curva $\gamma$ nel piano complesso.
L'unico polo (reale) presente all'interno del disco è $z_0:=\frac{1}{a}(1-\sqrt{1-a^2})$ e dunque $Res(f(z))_{z=z_0}=\lim_{z\rightarrow z_0}(z-z_0)f(z) = \frac{1}{z_0-\frac{1}{a}(1+\sqrt{1-a^2})}=-\frac{1}{\frac{2}{a}\sqrt{1-a^2}}$.
Allora si ha che $\oint_{\gamma}f(z)dz=2\pi i Res(f(z))=2\pi i(-\frac{1}{\frac{2}{a}\sqrt{1-a^2}})=-\frac{a\pi i}{\sqrt{1-a^2}}$.
Quindi$$
J=\frac{1}{2}\cdot(\frac{4}{ia}(-\frac{a\pi i}{\sqrt{1-a^2}}))=-\frac{2\pi}{\sqrt{1-a^2}}$$.
Dunque in definitiva si ha che:
$$
I=\frac{(r^2-1)\pi a}{r\sqrt{1-a^2}}
$$
Ovviamente non torna! Dovrei trovare una dipendenza da $\theta$. Dove me la sono persa?

Grazie in anticipo
Avatar utente
Isaac888
Average Member
Average Member
 
Messaggio: 329 di 599
Iscritto il: 18/03/2007, 21:29
Località: Pisa\Latiano

Re: Calcolo di un integrale

Messaggioda dissonance » 14/11/2022, 15:14

Si, ok, prima di iniziare: da dove viene questo integrale? C'è sicuramente qualche simmetria sferica dietro.

Comunque il problema è che la tua \(z\) dipende da \(\theta\) e da \(t\).
dissonance
Moderatore
Moderatore
 
Messaggio: 17206 di 27761
Iscritto il: 24/05/2008, 19:39
Località: Nomade

Re: Calcolo di un integrale

Messaggioda Isaac888 » 14/11/2022, 15:36

dissonance ha scritto:Si, ok, prima di iniziare: da dove viene questo integrale? C'è sicuramente qualche simmetria sferica dietro.

Questo integrale è lasciato come esercizio su un libro che sto leggendo. Viene da un problema di Dirichlet
$Delta u(x,y)=0$ nel disco $B(0,1)$
Le condizioni al bordo sono:
$u(x,y)={1\ \text{se}\ x^2+y^2=1, y>0; 0\ \text{se}\ x^2+y^2=1, y<0}$
Dunque la condizione al bordo la scrivo come $h(\theta)={1\ \text{se}\ \theta\in ]0,\pi[; 0\ \text{se}\ \theta \in]\pi,2\pi[}$
Per cui $$
u(r,\theta)=\frac{1}{2\pi}\int_{-\pi}^{\pi}{h(\phi)\frac{1-r^2}{1+r^2-2rcos(\theta - \phi)}d\phi}=\frac{1}{2\pi}\int_0^{\pi}{\frac{1-r^2}{1+r^2-2rcos(\theta - \phi)}d\phi}
$$
(Immagino tu lo sappia già ma il pezzo che voglio integrare si chiama nucleo integrale di Poisson. Solo che dipende da una differenza di angoli qui).
Il libro dà direttamente i valori al variare di $\theta$ ma lascia come esercizio il calcolo.
Il mio problema è intanto farlo in questo caso più facile. Vorrei accertarmi che l'unico modo per fare meno conti possibile sia usare il teorema dei residui. Soprattutto quando $h(\phi)$ non è costante.

dissonance ha scritto:Comunque il problema è che la tua \(z\) dipende da \(\theta\) e da \(t\).

A meno che non abbia sbagliato qualche cosa nel mio conto c'è modo di risolvere questo problema?
Avatar utente
Isaac888
Average Member
Average Member
 
Messaggio: 330 di 599
Iscritto il: 18/03/2007, 21:29
Località: Pisa\Latiano

Re: Calcolo di un integrale

Messaggioda pilloeffe » 16/11/2022, 23:37

Ciao Isaac888,

Credo che tu stia cercando di ricondurti all'integrale che compare in questo thread, che riporto qui di seguito per comodità:

$\int_0^{2\pi} 1/(a+bcos(t))\text{d}t = (2\pi)/\sqrt{a^2 - b^2} $

per $a > |b| $. Il fatto è che se si pone $t := \theta - \phi $, poi risulterebbe $\text{d}t = - \text{d}\phi $ ma soprattutto cambierebbero gli estremi di integrazione, che diventerebbero dipendenti da $\theta $:

$\int_0^{\pi} \frac{1}{1-a cos(\theta - \phi)}\text{d}\phi = - \int_{\theta}^{\theta - \pi} \frac{1}{1 - a cos(t)}\text{d}t = \int_{\theta - \pi}^{\theta} \frac{1}{1 - a cos(t)}\text{d}t $

Se non ho fatto male i conti, per $0 < a < 1 $ si ha:

$ \int \frac{1}{1-a \cdot cos(\theta-\phi)} \text{d}\phi = (2 arctan(((1 + a) tan((\phi - \theta)/2))/(sqrt(1 - a^2))))/\sqrt(1 - a^2) + c $

Pertanto si ha:

$\int_0^{\pi} \frac{1}{1-a cos(\theta - \phi)}\text{d}\phi = [(2 arctan(((1 + a) tan((\phi - \theta)/2))/(\sqrt(1 - a^2))))/\sqrt(1 - a^2)]_0^{\pi} = $
$ = (2 arctan(((1 + a) tan((\pi - \theta)/2))/(\sqrt(1 - a^2))) - 2 arctan(((1 + a) tan((- \theta)/2))/(\sqrt(1 - a^2))))/\sqrt(1 - a^2) = $
$ = 2/\sqrt(1 - a^2)[arctan(((1 + a) cot(\theta/2))/(\sqrt(1 - a^2))) + arctan(((1 + a) tan(\theta/2))/(\sqrt(1 - a^2)))] $

Da notare che se $\theta \to \pi $ si ottiene $\pi/\sqrt(1 - a^2) $
pilloeffe
Cannot live without
Cannot live without
 
Messaggio: 4991 di 10637
Iscritto il: 07/02/2017, 15:45
Località: La Maddalena - Modena


Torna a Analisi superiore

Chi c’è in linea

Visitano il forum: Nessuno e 1 ospite